What expression has the same value as -3/2-(2-3/8)+3/2

Answers

Answer 1

Answer:

[tex]\dfrac{-3}{2}-(2-\dfrac{3}{8})+\dfrac{3}{2}=\dfrac{-13}{8}[/tex]

Step-by-step explanation:

We need to find the value of expression [tex]\dfrac{-3}{2}-(2-\dfrac{3}{8})+\dfrac{3}{2}[/tex].

Firstly solving the second term as :

[tex](2-\dfrac{3}{8})=\dfrac{16-3}{8}=\dfrac{13}{8}[/tex]

Now the above expression becomes,

[tex]\dfrac{-3}{2}-(2-\dfrac{3}{8})+\dfrac{3}{2}\\=\dfrac{-3}{2}-\dfrac{13}{8}+\dfrac{3}{2}[/tex]

-3/2 and +3/2 equals 0.

It means that, [tex]\dfrac{-3}{2}-(2-\dfrac{3}{8})+\dfrac{3}{2}=\dfrac{-13}{8}[/tex]


Related Questions

What does the law of cosines reduce to when dealing with a right angle

Answers

Answer:

It is reduced to the equation of the Theorem of Pythagoras.

Step-by-step explanation:

Any triangle can be modelled by this formula under the Law of Cosine:

[tex]b = \sqrt{a^{2}+c^{2}-2\cdot a\cdot c\cdot \cos B}[/tex]

Where:

[tex]a[/tex], [tex]b[/tex], [tex]c[/tex] - Side lengths, dimensionless.

[tex]B[/tex] - Angle opposed to the side [tex]b[/tex], measured in sexagesimal degrees.

Now, let suppose that angle B is a right angle (90º), so that b is a hypotenuse and a and c are legs. Hence:

[tex]\cos B = 0[/tex]

And the equation is reduced to the form of the Theorem of Pythagoras, that is to say:

[tex]b = \sqrt{a^{2}+c^{2}}[/tex]

Astrid is in charge of building a new fleet of ships. Each ship requires 404040 tons of wood, and accommodates 300300300 sailors. She receives a delivery of 444 tons of wood each day. The deliveries can continue for 100100100 days at most, afterwards the weather is too bad to allow them. Overall, she wants to build enough ships to accommodate at least 210021002100 sailors.

Answers

To build the fleet of ships, Astrid must consider each of the given rates (i.e.  the daily tons of wood, the sailors per ship, etc.). The available deliveries are enough to build ships that can accommodate at least 2100 sailors.

Given that:

Required quantities

[tex]Wood = 40\ tons[/tex]

[tex]Sailors = 300[/tex] per ship

Available quantities

[tex]Wood = 4\ tons[/tex] daily

[tex]Days = 100[/tex] at most

First, we calculate the total tons of woods Astrid can receive.

[tex]Total = Days \times Wood\ Available[/tex]

[tex]Total = 100 \times 4[/tex]

[tex]Total = 400\ tons[/tex] ---- in 100 days

Next, we calculate the number of ships that can be made from the 400 tons.

[tex]Ships = \frac{Total\ tons}{Wood\ Required}[/tex]

So, we have:

[tex]Ships = \frac{400}{40}[/tex]

[tex]Ships = 10[/tex]

This means that Astrid can build up to 10 ships

The number of sailors the ship can accommodate is:

[tex]Sailors = Ships \times Sailors\ per\ ship[/tex]

So, we have:

[tex]Sailors = 10 \times 300[/tex]

[tex]Sailors = 3000[/tex]

It means the 10 ships can accommodate 3000 sailors.

3000 sailors is greater than 2100 sailors.

So, we can conclude that she can build enough ship for the 2100 sailors.

Read more about

https://brainly.com/question/17174491

Answer:

280 tons

Step-by-step explanation:

:)

In 2002, the population of a district was 22,800. With a continuous annual growth rate of approximately 5% what will the population be in 2012 according to the exponential growth function?

Answers

Answer:

37,139

Step-by-step explanation:

Given the following :

Population in 2002 = Initial population (P0) = 22,800

Growth rate (r) = 5% = 0.05

Growth in 2012 using the exponential growth function?

Time or period (t) = 2012 - 2002 = 10years

Exponential growth function:

P(t) = P0 * (1 + r) ^t

Where P(t) = population in t years

P(10) = 22800 * (1 + 0.05)^10

P(10) = 22800 * (1.05)^10

P(10) = 22800 * 1.62889

P(10) = 37138.797

P(10) = 37,139 ( to the nearest whole number)

Solve for x and draw a number line. 3x−91>−87 AND 17x−16>18

Answers

Answer:

I hope this will help!

Step-by-step explanation:

Manipulate the radius and height of the cone, setting different values for each. Record the radius, height, and exact volume of the cone (in terms of π). The first one has been done for you. Also calculate the decimal value of the volume, and verify that it matches the volume displayed by the tool. (You might see some discrepancies in the tool due to rounding of decimals.)

Answers

Answer:

The decimal value of the volume already given= 1885.2 unit³

For radius 11 unit height 12 unit

Volume= 484π unit³

Volume= 1520.73 unit ³

For radius 4 unit height 6 unit

Volume= 32π unit³

Volume= 100.544 unit³

For radius 20 unit height 15 unit

Volume= 2000π unit³

Volume= 6284 unit³

Step-by-step explanation:

The decimal value of the volume already given= 600π

The decimal value of the volume already given= 600*3.142

The decimal value of the volume already given= 1885.2 unit³

For radius 11 unit height 12 unit

Volume= πr²h/3

Volume= 11²*12/3 *π

Volume= 484π unit³

Volume= 1520.73 unit ³

For radius 4 unit height 6 unit

Volume = πr²h/3

Volume= 4²*6/3(π)

Volume= 32π unit³

Volume= 100.544 unit³

For radius 20 unit height 15 unit

Volume= πr²h/3

Volume= 20²*15/3(π)

Volume= 2000π unit³

Volume= 6284 unit³

Here's the right answer.

9(p−4)=−18 p= I am not great at math, please explain just a little bit

Answers

Answer:

[tex]\large \boxed{{p=2}}[/tex]

Step-by-step explanation:

9(p-4) = -18

Expand brackets.

9p -36 = -18

Add 36 on both sides.

9p -36 + 36 = -18 + 36

9p = 18

Divide both sides by 9.

(9p)/9 = 18/9

p = 2

Answer:

p = 2

Step-by-step explanation:

9(p - 4) = -18

You are solving for the variable, p. Note the equal sign, what you do to one side, you do to the other. Do the opposite of PEMDAS.

PEMDAS is the order of operation, and =

Parenthesis

Exponents (& Roots)

Multiplication

Division

Addition

Subtraction

~

First, divide 9 from both sides:

(9(p - 4))/9 = (-18)/9

(p - 4) = -18/9

p - 4 = -2

Isolate the variable, p. Add 4 to both sides:

p - 4 (+4) = -2 (+4)

p = -2 + 4

p = 4 - 2

p = 2

Check. Plug in 2 for p in the equation:

9(p - 4) = -18

9(2 - 4) = -18

9(-2) = - 18

-18 = -18.

~

what seven divided by 4

Answers

Answer:

7 divided by 4 is 1 ¾ as a fraction, or 1.75 as a decimal.

Step-by-step explanation:

Pls mark as brainliest answer

The calculated division of the numbers seven divided by 4 is 1 3/4

How to calculate the division of the numbers

From the question, we have the following parameters that can be used in our computation:

seven divided by 4

When represented as an equation, we have

seven divided by 4 = 7/4

Divide 7 by 4

So, we have the following result

seven divided by 4 = 1 3/4

Using the above as a guide, we have the following:

the result is 1 3/4

Read more about quotient at

brainly.com/question/11418015

#SPJ6

Describe how to solve an absolute value equation
*will give brainliest*

Answers

Answer:

Step 1: Isolate the absolute value expression.

Step2: Set the quantity inside the absolute value notation equal to + and - the quantity on the other side of the equation.

Step 3: Solve for the unknown in both equations.

Step 4: Check your answer analytically or graphically.

Step-by-step explanation:

Answer:

Rewrite the absolute value equation as two separate equations, one positive and the other negative

Solve each equation separately

After solving, substitute your answers back into original equation to verify that you solutions are valid

Write out the final solution or graph it as needed

Step-by-step explanation:

Events A and B are mutually exclusive. Find the missing probability.


P(A) = 1/4 P(B) = 13/20 P(A or B) = ?


4/5


1/2


9/10


3/8

Answers

Answer:

Option C.

Step-by-step explanation:

It is given that,

[tex]P(A)=\dfrac{1}{4}[/tex]

[tex]P(B)=\dfrac{13}{20}[/tex]

It is given that events A and B are mutually exclusive. It means they have no common elements.

[tex]P(A\cap B)=0[/tex]

We know that,

[tex]P(A\ or\ B)=P(A\cup B)=P(A)+P(B)-P(A\cap B)[/tex]

On substituting the values, we get

[tex]P(A\cup B)=\dfrac{1}{4}+\dfrac{13}{20}-0[/tex]

[tex]P(A\cup B)=\dfrac{5+13}{20}[/tex]

[tex]P(A\cup B)=\dfrac{18}{20}[/tex]

[tex]P(A\cup B)=\dfrac{9}{10}[/tex]

Therefore, the correct option is C.

The P (A or B) should be [tex]\frac{9}{10}[/tex]

Given that,

P(A) = 1 by 4  P(B) = 13 by 20

Based on the above information, the calculation is as follows:

[tex]= \frac{1}{4} + \frac{13}{20}\\\\= \frac{5+13}{20} \\\\= \frac{18}{20}\\\\= \frac{9}{10}[/tex]

Learn more: https://brainly.com/question/17429689?referrer=searchResults

Use the difference of squares identity to write this polynomial expression in factored form : 9x^2-49

Answers

Answer:

The expression in factored form is (3x - 7)(3x + 7)

Step-by-step explanation:

Here in this question, we are interested in using the difference of two squares to factor the given expression.

Mathematically, supposed we have two squares a^2 and b^2, and we are told to factorize a^2-b^2.

By using the difference of two squares;

a^2-b^2 can thus be written as;

(a-b)(a + b)

Now, we can apply same approach to the problem at hand.

9x^2 - 49

kindly note that 9x^2 can be written as ((3x)^2 and 49 can be written as 7^2

So applying what we have said earlier about difference of two squares;

9x^2 - 49 will be ;

(3x-7)(3x + 7)

Answer:

The answer is (3x - 7) (3x +7)

Step-by-step explanation:

Pick out the set of numbers that is not Pythagorean triple
9 40 46
16 30 34
10 24 26
50 120 130

Answers

Answer:

[tex]\huge\boxed{9,40,46}[/tex]

Step-by-step explanation:

Let's check it using Pythagorean Theorem:

[tex]c^2 = a^2 + b^2[/tex]

Where c is the longest sides, a and b are rest of the 2 sides

1) 9 , 40 , 46

=> [tex]c^2 = a^2 + b^2[/tex]

=> [tex]46^2 = 9^2 + 40^2[/tex]

=> 2116 = 81 + 1600

=> 2116 ≠ 1681

So, this is not a Pythagorean Triplet

2) 16, 30 and 34

=> [tex]c^2 = a^2 + b^2[/tex]

=> [tex]34^2 = 16^2 + 30^2[/tex]

=> 1156 = 256 + 900

=> 1156 = 1156

No need to check more as we've found the one which is not a Pythagorean Triplet.

Answer:

[tex] \boxed{ \huge{ \boxed{ \sf{ \blue{9 , \: 40 \:, 46 \: }}}}}[/tex]

Option A is the correct option.

Step-by-step explanation:

1. Let h , p and b are the hypotenuse , perpendicular and base of a right - angled triangle respectively.

From Pythagoras theorem,

[tex] \sf{ {h}^{2} = {p}^{2} + {b}^{2} }[/tex]

Here, we know that the hypotenuse is always greater than perpendicular and base,

h = 46 , p = 40 , b = 9

⇒[tex] \sf{ {46}^{2} = {40}^{2} + {9}^{2} }[/tex]

⇒[tex]2116 = 1600 + 81[/tex]

⇒[tex] \sf{2116  ≠ 1681}[/tex]

Thus , the relation [tex] \sf{ {h}^{2} = {p}^{2} + {b}^{2} }[/tex] is not satisfied by h = 46 , p = 40 , b = 9

So, The set of numbers 9 , 40 , 46 is not Pythagorean triple.

------------------------------------------------------

2. 16 , 30 , 34

h = 34 , p = 30 , b = 16

[tex] \sf{ {h}^{2} = {p}^{2} + {b}^{2} }[/tex]

⇒[tex] \sf{ {34}^{2} = {30}^{2} + {16}^{2} }[/tex]

⇒[tex] \sf{1156 = 900 + 256}[/tex]

⇒[tex] \sf{1156 = 1156}[/tex]

The relation [tex] \sf{ {h}^{2} = {p}^{2} + {b}^{2} }[/tex] is satisfied by the particular values of h , p and b i.e h = 34 , p = 30 , b = 16

So, the set of numbers 16 , 30 , 34 is a Pythagorean triple.

------------------------------------------------------

3. 10, 24 , 26

h = 26 , p = 24 , b = 10

[tex] \sf{ {h}^{2} = {p}^{2} + {b}^{2} }[/tex]

⇒[tex] \sf{ {26}^{2} = {24}^{2} + {10}^{2} }[/tex]

⇒[tex] \sf{676 = 576 + 100}[/tex]

⇒[tex] \sf{676 = 676}[/tex]

The relation [tex] \sf{ {h}^{2} = {p}^{2} + {b}^{2} }[/tex] is satisfied by the particular values of h , p and h i.e h = 26 , p = 24 , b = 10

So, the set of numbers 10, 24 , 26 is the Pythagorean triple.

-----------------------------------------------------

4. 50 , 120 , 130

h = 130 , p = 120 , b = 50

[tex] \sf{ {h}^{2} = {p}^{2} + {b}^{2} }[/tex]

⇒[tex] \sf{ {130}^{2} = {120}^{2} + {50}^{2} }[/tex]

⇒[tex] \sf{16900 = 14400 + 2500}[/tex]

⇒[tex] \sf{16900 = 16900}[/tex]

The relation [tex] \sf{ {h}^{2} = {p}^{2} + {b}^{2} }[/tex] is satisfied by the particular values of h , p and b i.e h = 130 , p = 120 , b = 50

So, the set of numbers 50, 120 , 130 is the Pythagorean triple.

-----------------------------------------------------

In this way, to satisfy the Pythagoras Theorem , the hypotenuse ( h ) , perpendicular ( p ) and the base ( b ) of a right - angles triangle should have the particular values in order. These values of h , p and b are called Pythagorean triple.

Hope I helped!

Best regards!!

(x+3)(x-5)=(x+3)(x−5)=

Answers

Answer:

All real numbers are solutions. 0=0

Step-by-step explanation:

(x+3)(x−5)=(x+3)(x−5)

Step 1: Simplify both sides of the equation.

x2−2x−15=x2−2x−15

Step 2: Subtract x^2 from both sides.

x2−2x−15−x2=x2−2x−15−x2

−2x−15=−2x−15

Step 3: Add 2x to both sides.

−2x−15+2x=−2x−15+2x

−15=−15

Step 4: Add 15 to both sides.

−15+15=−15+15

0=0

All real numbers are solutions.

For a ,a relationship to be a function, which values cannot repeat: the x-
values or the y-values? *

Answers

Answer:

              The  x - values

The y-values repeat in various functions (for example: quadratic function: y=x²; y=4 for x=2 and for x=-2)  

Translate the following phrase into an algebraic expression using the variable m. Do not simplify,
the cost of renting a car for one day and driving m miles if the rate is $39 per day plus 45 cents per mile

Answers

Answer:

y  = 0.45X + 39  

Find the volume of a pyramid with a square base, where the side length of the base is 17 in 17 in and the height of the pyramid is 9 in 9 in. Round your answer to the nearest tenth of a cubic inch.

Answers

Answer:

The volume of the pyramid is 867 inch^3

Step-by-step explanation:

Here in this question, we are interested in calculating the volume of a square based pyramid.

Mathematically, we can use the formula below to calculate the volume V of a square based pyramid.

V = a^2h/3

where a represents the length of the side of the square and h is the height of the pyramid

From the question, the length of the side of the square is 17 in while the height is 9 in

Plugging these values, we have ;

V = (17^2 * 9)/3 = 17^2 * 3 = 867 cubic inch

Raj tested his new flashlight by shining it on his bedroom wall. The beam of light can be described by the equation . How many inches wide is the beam of light on the wall?

Answers

Answer:

12 inches

Step-by-step explanation:

Raj tested his new flashlight by shinning it on his bedroom wall the beam of the light can be described by the equation (x^2-2x) + (y^2-4y) - 31=0. how many inches wide is the beam of light on the wall

Solution

Given:

(x^2-2x) + (y^2-4y) - 31=0

By completing the square

(x^2-2x) + (y^2-4y) - 31=0

(x^2-2x+1-1) + (y^2-4y+4-4)-31=0

(x-1)^2 -1 + (y-2)^2 - 4 - 31=0

(x-1)^2 + (y-2)^2 - 1 - 4 - 31=0

(x-1)^2 + (y-2)^2 - 36=0

(x-1)^2 + (y-2)^2=36

Writing the equation in the form: (x-h)^2+(y-k)^2=r^2

(x-1)^2+(y-2)^2=6^2

From the above, r=6

Where,

r=radius

how wide is the diameter ?

radius=6

Diameter= 2 × radius

=2×6

=12 inches

Answer:

12

Step-by-step explanation:

to graph it just scan the equation on photo math!!

Calculate JK if LJ = 14, JM = 48, and LM = 50

Answers

Answer:

JK = 6.86

Step-by-step explanation:

The parameters given are;

LJ = 14

JM = 48

LM = 50

[tex]tan(\angle JML )= \dfrac{Opposite \ leg \ length}{Adjacent \ leg \ length} = \dfrac{LK}{JM} = \dfrac{14}{48} = \dfrac{7}{24}[/tex]

[tex]tan \left( \dfrac{7}{24} \right)= 16.26 ^{\circ }[/tex]

∠JML = 16.26°

Given that ∠JML is bisected by KM, we apply the angle bisector theorem which states that a ray that bisects an interior angle of a triangle bisects the opposite (bisected angle facing side) in the proportion of the ration of the other two sides of the triangle.

From the angle bisector theorem, we have;

LM/JM = LK/JK

50/48 = LK/JK................(1)

LK + KJ = 14.....................(2)

From equation (1), we have;

LK = 25/24×JK

25/24×KJ + JK = 14

JK×(25/24 + 1) = 14

JK × 49/24 = 14

JK = 14×24/49 = 48/7. = 6.86.

JK = 6.86

ASAP PLEASE GIVE CORRECT ANSWER
In a rectangular coordinate system, what is the number of units in the distance from the origin to the point $(-15, 8)$? Enter your answer

Answers

distance of a point [tex](x,y)[/tex] from origin is $\sqrt{x^2+y^2}$

so distance is $\sqrt{(-15)^2+(8)^2}=\sqrt{225+64}=\sqrt{289}=17$

Answer:

Distance=17 units

Step-by-step explanation:

Coordinates of the origin: (0, 0)

Coordinates of the point in question: (-15, 8)

Distance formula for any two points [tex](x_1,y_1), (x_2,y_2)[/tex] on the plane:

[tex]distance=\sqrt{(x_2-x_1)^2+(y_2-y_1)^2} \\distance=\sqrt{(-15-0)^2+(8-0)^2}\\distance=\sqrt{(15)^2+(8)^2}\\distance=\sqrt{225+64} \\distance=\sqrt{289} \\distance=17[/tex]

ABC is an equilateral triangle, solve y

Answers

Answer:

y is 60⁰

because all sides are equal

Answer:

60 degrees

Step-by-step explanation:

In an equilateral triangle, the angles are equiangluar and the sides are equal.

180 degrees in a triangle/3 sides =

= 60 degrees per side

Please help quickly!!
A truck is driving up a hill with a 24% grade, so it climbs 24 feet vertically for every 100 feet horizontally.
What is the slope of the hill?

Answers

Answer:

6/25

Step-by-step explanation:

rise / run

24/100 = 6/25

Answer:

[tex]\frac{6}{25}[/tex]

Step-by-step explanation:

The slope of any relationship is always rise over run. This means the vertical distance traveled over the horizontal distance traveled will get us our slope.

We travels 24 feet vertically for every 100 feet horizontally, so:

[tex]\frac{24}{100}[/tex].

We can simplify this fraction to find the slope in fraction form.

[tex]\frac{24\div4}{100\div4} = \frac{6}{25}[/tex]

So the slope of this equation is [tex]\frac{6}{25}[/tex].

Hope this helped!

PLEASE ANSWER QUICKLY ASAP
COMPLETE QUESTION B​

Answers

Answer:

Sector

Step-by-step explanation:

A sector of a circle is the portion of circle enclosed by two radii and arc

What are the dimensions of the matrix?

Answers

The order of a matrix is m×n where m is the number of rows and n is the number of columns.

can you count and find what are m and n here?

Answer:

Step-by-step explanation:

Number of rows X Number of columns

Rows = 3

Columns = 2

answer = 3x2

Sandy’s older sister was given $2,400 and was told to keep the balance of the money after sharing with her siblings. Give Sandy exactly $350. Write Sandy’s portion

Answers

Sandy got 350 out of 2400.

Her portion is 350/2400 which can be reduced to:

35/240 = 7/48

The portion is 7/48

A company makes nylon and canvas backpacks. The profit on a nylon backpack is $3 and the profit on a canvas backpack is $10. How many backpacks must the company sell to make a profit of more than $250? Write a linear inequality that describes the situation.

Answers

Answer:

3x +10y  is greater than or equal to 250.

Step-by-step explanation:

The question asks us to write an inequality which shows that both nylon and canvas added should be greater than or equal to 250.

Since we don't know the number of nylon backpacks and canvas backpacks the company makes, we used the variables "x" and "y" to represent the number of backpacks they made from each style.

Answer:

3n + 10c > 250

Step-by-step explanation:

I confirmed it in grandpoint


* Graph these numbers on a number line.
-5,3, -2,1
-5

Answers

-5,3,-2,1 on a number line

<-|----|----|----|----|----|----|----|----|->

-5 -2 0 1 3

n urn contains 3 red balls, 9 green, 2 yellow, 2 orange, and 4 purple balls. Two balls aredrawn, one at a time with replacement. Find the probability of drawing a green ball and an orangeball.

Answers

Answer:

[tex]\frac{9}{100}[/tex]

Step-by-step explanation:

Given:

Number of red balls, n(R) = 3

Number of green balls, n(G) = 9

Number of yellow balls, n(Y) = 2

Number of orange balls, n(O) = 2

Number of purple balls, n(P) = 4

Two balls are drawn one at a time with replacement.

To find:

Probability of drawing a green ball and an orange ball ?

Solution:

Total number of balls, n(Total) = 3 + 9 + 2 + 2 + 4 = 20

Formula for probability of an event E is given as:

[tex]P(E) = \dfrac{\text{Number of favorable cases}}{\text {Total number of cases}}[/tex]

Probability that a green ball is drawn at first:

[tex]P(Green) = \dfrac{\text{Number of Green balls}}{\text {Total number of Balls}}[/tex]

[tex]P(Green) = \dfrac{9}{20}[/tex]

Now, the ball is replaced , so total number of balls remain the same i.e. 20.[tex]P(Orange) = \dfrac{\text{Number of Orange balls}}{\text {Total number of Balls}}[/tex]

[tex]P(Orange) = \dfrac{2}{20} = \dfrac{1}{10}[/tex]

[tex]P(Green\ then\ orange) = P(Green) \times P(Orange)\\\Rightarrow P(Green\ then\ orange) = \dfrac{9}{10} \times \dfrac{1}{10}\\\Rightarrow P(Green\ then\ orange) = \bold{ \dfrac{9}{100} }[/tex]

john always wears a shirt, pants, socks, and shoes. he owns 12 pairs of socks, 3 pairs of shoes, 5 pairs of pants, and 5 shirts. how many different outfits can john make? PLEASE ANSWER

Answers

Answer:

900 outfits

Step-by-step explanation:

You just have to multiply them all together :)

I will give brainliest to the right answer!! Find the vertex and the length of the latus rectum. x= 1/2 (y - 5)² + 7

Answers

Answer:

(7, 5)2

Step-by-step explanation:

When the quadratic is written in vertex form:

  x = a(y -k)^2 +h

the vertex is (x, y) = (h, k), and the length of the latus rectum is 1/a.

For your given equation, ...

  x = (1/2)(y -5)^2 +7

you have a=1/2, k = 5, h = 7, so ...

  the vertex is (7, 5)

  the length of the latus rectum is 1/(1/2) = 2

Find the value of x. Round to the nearest tenth.

15.9
12.4
12.8
16.3

Answers

Answer:

x = 15.9

Step-by-step explanation:

Since this is a right triangle, we can use trig functions

cos theta = adj / hyp

cos 28 = 14/x

x  cos 28 = 14

x = 14 / cos 28

x=15.85598

Rounding to the nearest tenth

x = 15.9

Simplify the following expression.

Answers

Answer:

3x+11y-3

Step-by-step explanation:

Hey! So here is what you do to solve the problem-

Combine like terms:

(x) 5x-2x=3x

(y) 3y+8y=11y

(#) 7-10 =-3

So....

3x+11y-3 is your answer!

Hope this helps!:)

Other Questions
Which number is equal to 10^-3?-1,000 -300.0010.003 Over a long period of time, a group of people were exposed to high degrees of UV radiation from the sun. While no one became sick immediately, doctors did predict that those exposed had an increased risk of cancer in skin cells. Which type of mutation will this exposure cause? A. acquired, harmful B. acquired, neutral C. inherited, beneficial D. inherited, neutral 5 times the quantity 7 minus a number f in an algebraic expression. Which of the following is TRUE of the organ and tissue donation designation process? You may change your designated preference at any time online. Paramedics determine what organs can be donated while at the scene of a collision. The donor registry is located at and managed by the local library. Surviving family members must consent to organ donation. Slack Inc. borrowed $400,000 on April 1. The note requires interest at 12% and principal to be paid in one year. How much interest is recognized for the period from April 1 to December 31? a. $0. b. $48,000. c. $32,000. d. $36,000 How did humanism impact political thinking during the Renaissance? It allowed emperors to justify conquering new territories. It inspired merchants to spread European ideas about democracy. It led scholars to support religious leaders over independent monarchs. It reduced the importance of religion over how people thought about society. Find the slope of the line passing through the points (-9, -3) and (7,-7).DIDDUndefined$? PLEASE HELP! 10 POINTS Which line would be the line of best fit for the scatter plot? What collateral secures a mortgage?O A.5% of the purchase price of the houseB.the house itselfC.10% of the purchase price of the houseD.none "Rihanna Company is considering purchasing new equipment for $379,200. It is expected that the equipment will produce net annual cash flows of $48,000 over its 10-year useful life. Annual depreciation will be $37,920. Compute the cash payback period. (Round answer to 1 decimal place, e.g. 10.5.)" Advantages of using a resource person in handling the first aid lesson Do some online research on the characteristics of an orchestral performance. Write a short paragraph describing the musicians, instruments, audience, and conductor. Then, write a second paragraph about the differences that you observe between an orchestral performance and a rock concert. Explain which of the two you prefer. when was the declaration of Independence signed Examine the table of input (x) and output (y) values below. What is the relationship between the input and output values? Write an equation for this relationship. input (x) 1 0 1 2 3 4 5 output (y) 4 1 2 5 8 11 14 What do you notice about the relationship between the x and y values? What is the (10) for? Janet is signing up for college classes and she wants to study world languages. Which profession would give her the opportunity to use the languages she studied? Name a continent on this meridian-15 degrees East Derrick Iverson is a divisional manager for Holston Company. His annual pay raises are largely determined by his divisions return on investment (ROI), which has been above 20% each of the last three years. Derrick is considering a capital budgeting project that would require a $3,080,000 investment in equipment with a useful life of five years and no salvage value. Holston Companys discount rate is 17%. The project would provide net operating income each year for five years as follows:Sales $3,400,000Variable expenses 1,450,000Contribution margin 1,950,000Fixed expenses: Advertising, salaries, and other fixedout-of-pocket costs $670,000 Depreciation 828,000 Total fixed expenses 1,498,000Net operating income $452,000Required:a. Compute the project's net present value.b. Compute the project's simple rate of return.c. Would the company want Derrick to pursue this investment opportunity?d. Would Derrick be inclined to pursue this investment opportunity? There are 30 colored marbles inside a bag. Six marbles are yellow, 9 are red, 7 are white, and 8 are blue. One is drawn at random. Which color is most likely to be chosen? A. white B. red C. blue D. yellow Include ALL work please! If the domino effect occurs as a result of changes in the money supply, what will most likely happen as an immediate result of interest rates being increased? Borrowing will decrease. Investing will decrease. Inflation will increase. Liquidity will increase.